1
$\begingroup$

We let $X \sim \mu$ and $Y \sim \nu$ be the source space and target space for optimal transport. And we let $c: X \times Y \rightarrow \mathbb{R} \cup \{\infty\}$ be a cost function. We recall that $\pi \in \prod(\mu,\nu)$ is a coupling of $\mu,\nu$, if $\text{proj})_{\#}^{X}(\pi) = \mu, \text{proj}_{\#}^{Y}(\pi) = \nu$.
We define $$\delta_{\prod(\mu,\nu)}(\pi) = \begin{cases} 0, & \text{ if } \pi \in \prod(\mu,\nu)\\ \infty, &\text{ otherwise }\end{cases}.$$ We want to show $$\sup_{\varphi,\psi} \int \varphi d\mu + \int \psi d\nu - \int \varphi d\pi - \int \psi d\pi = \delta_{\pi (\mu,\nu)}(\pi),$$ where $\varphi$ and $\psi$ satisfy the constraint $\varphi(x) + \psi(y) \leq c(x,y)$ for all $(x,y) \in X \times Y$.
My proof so far: We first consider the case when $\pi \in \prod(\mu,\nu)$. When $\pi \in \prod(\mu,\nu)$, we have $\delta_{\prod(\mu,\nu)}(\pi) = 0$ by construction. This means that we need to show $\sup_{\varphi,\psi} \int \varphi d\mu + \int \psi d\nu - \int \varphi d\pi - \int \psi d\pi = 0$.
We define $\varphi \oplus \psi$ be a function on $X \times Y$ by $(\varphi \oplus \psi)(x,y) = \varphi(x) + \psi(y)$. Then if $\pi \in \prod(\mu,\nu)$, we have $$\int (\varphi \oplus \psi) d\pi = \int \varphi(x) d\pi (x,y) + \int \psi (y) d\pi (x,y) = \int \varphi d\mu + \int \psi d\nu.$$ So we have when $\pi \in \prod(\mu,\nu)$, $$\int \varphi d\mu + \int \psi d\nu - \int (\varphi + \psi)d\pi = 0.$$ So when $\pi \in \prod(\mu,\nu)$, we have $\sup_{\varphi,\psi} \int \varphi d\mu + \int \psi d\nu - \int \varphi d \pi - \int \psi d\pi = 0$.
However, I’m not sure how to deal with the case when $\pi \notin \prod(\mu,\nu)$.

$\endgroup$
3
  • $\begingroup$ What have you tired in case $\pi \not\in \Pi$? $\endgroup$ Commented Jul 1 at 6:55
  • $\begingroup$ @gerw I’m thinking if I could use the Hahn-Banach theorem. But I’m not sure $\endgroup$ Commented Jul 1 at 8:11
  • $\begingroup$ For $\pi \not\in \Pi$, there are three possible reasons. In each of these cases, you can check that the supremum is $+\infty$ by constructing suitable $\varphi, \psi$. $\endgroup$ Commented Jul 1 at 9:10

0

You must log in to answer this question.

Start asking to get answers

Find the answer to your question by asking.

Ask question

Explore related questions

See similar questions with these tags.